Find the value of each variable in the parallelogram.lg + 4) 7.16- h65°g=h=

Find The Value Of Each Variable In The Parallelogram.lg + 4) 7.16- H65g=h=

Answers

Answer 1

h = 9

g = 61°

Explanation:

The opposite sides of a parallelogram are parallel and congruent.

This means the opposite sides are equal

equating the sides:

7 = 16 - h

subtract 16 from both sides:

7 - 16 = - h

-9 = -h

divide both sides by -1

-9/-1 = -h/-1

h = 9

The opposite angles of a parallelogram are equal

equating the angles:

(g + 4)° = 65°

g + 4 = 65

subtract 4 from both sides:

g + 4 -4 = 65 -4

g = 61°


Related Questions

can you help me please

Answers

If the value of x increases and the value of y also increases then we can say that the slope is Increases,

If the value of x increases but the value of y decreases then we can say that the slope is decreases.

In the given coordinates of tempreature and icecreams sold is

as the value of Tempreature increases from 60 to 80, the number of icecreams (value of y) is also increases,

but at tempreature =90 the number of icecreams is 20,

thus, the value of y is decreases at x=90

Hence slope changes sign

first it gets positive from tempreature (0 to 80) and then decreases to 90

Answer : C) Slope changes sign

Questionf(2)Find R(2) where f(2)g(2)x² – a- 2 - 3010x + 100and g(2)-22 – 5x + 6611x + 110(Simplify your answer.)Provide your answer below:

Answers

Answer:

Given that,

To find,

[tex]R(x)=\frac{f(x)}{g(x)}[/tex]

where,

[tex]f(x)=\frac{x^2-x-30}{10x+100}[/tex][tex]g(x)=\frac{-x^2-5x+66}{11x+110}[/tex]

Simplifing f(x) and g(x), we get

[tex]f(x)=\frac{x^2-x-30}{10x+10)}=\frac{x^2-6x+5x-30}{10(x+10)}[/tex][tex]=\frac{x(x-6)+5(x-6)}{10(x+10)}=\frac{(x-6)(x+5)}{10(x+10)}[/tex][tex]f(x)=\frac{(x-6)(x+5)}{10(x+10)}-----(1)[/tex]

This is the simplified form of f(x).

For g(x) we get,

[tex]g(x)=\frac{-x^2-5x+66}{11x+110}=\frac{x^2+5x-66}{-11(x+10)}[/tex][tex]=\frac{x^2+11x-6x-66}{-11(x+10)}=\frac{x(x+11)-6(x+11)}{-11(x+10)}[/tex][tex]g(x)=\frac{(x+11)(x-6)}{-11(x+10)}------(2)[/tex]

[tex]\frac{i}{g(x)}=\frac{-11(x+10)}{(x+11)(x-6)}[/tex]

Now To find R(x), we get

[tex]R(x)=\frac{f(x)}{g(x)}=f(x)\times\frac{1}{g(x)}[/tex][tex]=\frac{(x-6)(x+5)}{10(x+10)}\times\frac{-11(x+10)}{(x+11)(x-6)}[/tex][tex]=\frac{-11(x+5)}{10(x+11)}[/tex]

we get,

[tex]R(x)=\frac{-11(x+5)}{10(x+11)}[/tex]

Answer is:

[tex]R(x)=\frac{-11(x+5)}{10(x+11)}[/tex]

how to write this on a number line1 plus x less than 5

Answers

We are given the following inequality

[tex]1+x<5[/tex]

Let us first solve the inequality for x.

Subtract 1 from both sides of the inequality

[tex]\begin{gathered} -1+1+x<5-1 \\ x<4 \end{gathered}[/tex]

So the solution of the inequality is all the values of x less than 4

Now let us graph this solution on a number line.

Find the coordinates of the missing vertex of rectangle ABCD with A(-3, 3),B(5, 3), and D(-3, -1).O (5, -1)© (-11, 7)O (5,3)O (1, -1)

Answers

Given:

The given vertex of a rectangle ABCD are A=(-3,3), B=(5,3) and D=(-3,-1)

To find: Missing vertex, that means vertex C

The graph is as follows:

From the above graph, the coordinates of point C are (5,-1).

Hence, the required answer is (5,-1).

Write the standard form of the quadratic function f(x) whose graph has vertex (1,2) and passes through (2,4)

Answers

Step 1. We are given the vertex of the quadratic function:

[tex](1,2)[/tex]

And a point:

[tex](2,4)[/tex]

Required: Find the standard form of the quadratic equation.

Step 2. Since we know the vertex of the quadratic function we will start by using the vertex form of the quadratic function:

[tex]y=a(x-h)^2+k[/tex]

Where (h, k) is the vertex, in this case:

[tex]\begin{gathered} h=1 \\ k=2 \end{gathered}[/tex]

Step 3. To use the previous equation

[tex]y=a(x-h)^{2}+k[/tex]

We will need to find the value of a. For that, we substitute the h and k values:

[tex]y=a(x-1)^2+2[/tex]

And as the values of x and y, we substitute the values of the given point (2,4) where x=2 and y=4

[tex]4=a(2-1)^2+2[/tex]

Solving for a:

[tex]\begin{gathered} 4-2=a(1)^2 \\ 2=a(1) \\ 2=a \end{gathered}[/tex]

Step 4. Now that we know that the value of a is 2, we go back to our general equation:

[tex]y=a(x-h)^{2}+k[/tex]

Substitute the value of a, h, and k:

[tex]y=2(x-1)^2+2[/tex]

This is the equation in the vertex form, but we need it in standard form.

Step 5. The standard form of the quadratic function is:

[tex]f(x)=ax^2+bx+c[/tex]

To convert our equation into the standard form, first, we change y by f(x):

[tex]\begin{gathered} y=2(x-1)^{2}+2 \\ \downarrow \\ f(x)=2(x-1)^2+2 \end{gathered}[/tex]

Then, we use this formula for the binomial squared:

[tex](a-b)^2=a^2-2ab+b^2[/tex]

The result is:

[tex]f(x)=2(x^2-2x+1)+2[/tex]

Simplifying:

[tex]\begin{gathered} f(x)=2x^2-4x+2+2 \\ \downarrow \\ \boxed{f\mleft(x\mright)=2x^2-4x+4} \end{gathered}[/tex]

That is the standard form of the quadratic function.

Answer:

[tex]\boxed{f(x)=2x^{2}-4x+4}[/tex]

y=-x^2-4x-8Identify the vertex, the axis of symmetry, the maximum or minimum value, and the range of the parabola.

Answers

Here we have the following parabola:

[tex]y=-x^2-4x-8[/tex]

To find the vertex, we could use the following formula:

[tex]V(x,y)=V(\frac{-b}{2a},\frac{-b^2}{4a}+c)[/tex]

Where a, b and c are the coefficients of the quadratic function:

[tex]y=ax^2+bx+c[/tex]

As you can see, in this problem a = -1 , b = -4 and c = -8. Thus,

[tex]V(x,y)=V(\frac{-(-4)}{2(-1)},\frac{-(-4)^2}{4(-1)}-8)[/tex]

This is:

[tex]V(-2,-4)[/tex]

Then, the vertex of the parabola is (-2,-4)

The axis of symmetry of the parabola is the line x=-2. Since the vertex is situated at the coordinates (-2,-4), that means that the parabola is symmetrical around this line.

The vertex is maximum point of the parabola.

The range, is defined as all the values that the y-axis could take. If we notice, that is:

[tex](-\infty,-4\rbrack[/tex]

I'm going to upload a picture of the parabola:

Question 3 Olivia is making pancakes for breakfast. The recipe calls for 0.5 quart of milk and 2.5 cups of flour. She has quart of 3/8 quart of milk and 18/8 cups of flour. Olivia makes the recipe with the milk and flour she has. Explain her error. Hint: Convert all of them to either decimal or fraction so that you can compare the values. Challenge: How much more or less milk does Olivia need? How much more or less flour does Olivia need?

Answers

ANSWER and EXPLANATION

The recipe calls for 0.5 quart of milk and 2.5 cups of flour.

She has 3/8 quart of milk and 18/8 cups of flour.

To know the error she made, we have to find the ratio of milk to flour in the recipe and the ratio of milk to flour that she used and see if the ratios are the same.

RECIPE

Ratio of milk to flour is:

[tex]\begin{gathered} 0.5\text{ : 2.5} \\ \Rightarrow\text{ 1 : 5} \end{gathered}[/tex]

That is the ratio of milk to flour.

USED BY OLIVIA

Ratio of milk and flour that she used is:

[tex]\begin{gathered} \frac{3}{8}\text{ : }\frac{18}{8} \\ =>\text{ 3 : 18} \\ \Rightarrow\text{ 1 : 6} \end{gathered}[/tex]

We can already see that the ratios are not the same.

By comparing the ratios, we see that she actually used more flour than she was supposed to.

Which of the following choices are equivalent to the expression below? Check all that applyx^(3/8)

Answers

Answer: [tex]\begin{gathered} \text{\lparen}\sqrt[8]{x^3}\text{ \rparen \lparen option B\rparen} \\ \\ \text{\lparen}\sqrt[8]{x^\text{\rparen}^3}\text{ \lparen option D\rparen} \\ \\ (x^3)\placeholder{⬚}^{\frac{1}{8}}\text{ \lparen option F\rparen} \end{gathered}[/tex]

Explanation:

Given:

[tex]x^{\frac{3}{8}}[/tex]

To find:

the equivalence of the given expression

[tex]\begin{gathered} We\text{ will apply exponent rule:} \\ x^{\frac{1}{b}}\text{ = }\sqrt[b]{x} \\ x^{\frac{a}{b}}\text{ = \lparen}\sqrt[b]{x})\placeholder{⬚}^a \\ \\ Applying\text{ same rule to the given expression:} \\ x^{\frac{3}{8}}\text{ = \lparen}\sqrt[8]{x})\placeholder{⬚}^3 \end{gathered}[/tex][tex]\begin{gathered} (\sqrt[8]{x})\placeholder{⬚}^3\text{ can also be written as \lparen}\sqrt[8]{x^3}) \\ x^{\frac{3}{8}}=\text{ \lparen}\sqrt[8]{x^3}\text{ \rparen} \end{gathered}[/tex][tex]\begin{gathered} from\text{ \lparen}\sqrt[8]{x})\placeholder{⬚}^3,\text{ }\sqrt[8]{x}\text{ = x}^{\frac{1}{8}} \\ \\ (\sqrt[8]{x})\placeholder{⬚}^3\text{ = \lparen x}^{\frac{1}{8}})\placeholder{⬚}^3 \\ =(\text{x}^3)\placeholder{⬚}^{\frac{1}{8}} \end{gathered}[/tex]

is the prime factorization of what composite number? 91 point

Answers

Answer: The question isn't clarified, what are you looking for?

Step-by-step explanation:

Answer:

The Prime Factors of 91 are 1, 7, 13, 91

Step-by-step explanation:

Match each function with its graph 1. f (x) =x³+3x²2. f (x) = -x (x-1) (x+2)3. f (x) = -x³+3x²4. f (x) = x (x+1) (x-2)

Answers

to understand this graphs you must find the roots on each of the functions.

start by funtion 1.

[tex]\begin{gathered} x^3+3x^2=0 \\ x\cdot(x^2+3x)=0 \\ x=0 \\ (x^2+3x)=0 \\ x(x+3)=0 \\ x=0 \\ x+3=0 \\ x=-3 \end{gathered}[/tex]

for function 1 you will need to find a graph that only intercept the x-axis on 0 an -3. In this case it will be the graph A.

Do the same for each function

[tex]\begin{gathered} -x\cdot(x-1)\cdot(x+2) \\ x=0 \\ x-1=0 \\ x=1 \\ x+2=0 \\ x=-2 \end{gathered}[/tex]

function 2, the interceptions are 0,1 and -2. Graph C will be the correct one for this function

function 3

[tex]\begin{gathered} -x^3+3x^2=0 \\ x\cdot(-x^2+3x)=0 \\ x=0 \\ (-x^2+3x)=0 \\ x(-x+3)=0 \\ x=0 \\ -x+3=0 \\ x=3 \end{gathered}[/tex]

for fuction 3, roots will be 0 and 3, the associated graph will be D

and lastly the roots for function 4.

[tex]\begin{gathered} -x\cdot(x+1)\cdot(x-2) \\ x=0 \\ x+1=0 \\ x=-1 \\ x-2=0 \\ x=2 \end{gathered}[/tex]

The associated graph is B.

Should Jenna buy the smart phone at top quality or big value? support your answer with a mathematical evidence. Assume that getting the lowest price is Jenna's only consideration.?

Answers

Let the price of the item be t as stated in the question.

This means if Top quality is selling them at 15% off the list price, then the new price can be represented as;

(A)

[tex]\begin{gathered} Price=t-discount \\ \text{Price}=t-(t\times0.15) \\ \text{Price}=t-0.15t---(1) \\ \text{The second expression that can be used to }represent\text{ the discounted price is;} \\ \text{Price}=(t-0.15t) \\ \text{Price}=0.85t---(2) \end{gathered}[/tex]

(B)

Equation 1 shows the original list price less the discounted amount (which is 15 percent times the list price, t). The result is the price now paid eventually

Equation 2 shows the percentage of the list price that would be paid by Jenna after deducting the discount, which means she would be paying 85 percent of the list price (that is 0.85)

(C)

A smartphone on sale at 1/4 off its list price, would also mean its being sold at a discount of 25%. One-quarters of 100 percent would be 25, hence the smartphone is at 25% off the list price.

However, where the phone is being sold at 75% of its list price means, the list price now has 25% taken off. That is, the price at Big Value is

[tex]\begin{gathered} \text{Price}=0.75t \\ \text{Discount=t-0.75t} \\ \text{Discount}=0.25 \end{gathered}[/tex]

That means the discount at Big value is 25% (or 0.25)

The discount at Top Quality is 25% (0.25 or 1/4)

Jenna can buy at either of the store. Since she is already at Top Quality, she can just go ahead and buy it right there

Function f(x) = |x| is transformed to create function g(x) = |x - 7| + 2.What transformations are performed to function f to get function g?Select each correct answer.Function f is translated 7 units to the left.Function f is translated 7 units down.Function f is translated 2 units up.Function f is translated 7 units to the right.Function f is translated 7 units upFunction f is translated 2 units to the right.Function f is translated 2 units to the left.Function f is translated 2 units down.

Answers

We will have the following:

*Function f is translated 7 units rigth.

*Function f is translated 2 units up.

triangle W quadrilateral hexagon pentagon 2 + please help me out with thisWhat is a name for this shapetrianglequadrilateralhexagonpentagon

Answers

To determine the name of the shape you have to count its sides.

If it has 3 sides is a triangle, if it has 4 sides is a quadrilateral, if it has 5 sides is a pentagon and if it has 6 sides is a hexagon,

The shape has 6 sides so it is a hexagon.

Which equation is the inverse of y = 16x2 + 1?O y=+T6x-1O y =16+Oy AVTOINX-1Оy=

Answers

We need to find the inverse function of:

[tex]undefined[/tex]

please help I don't understand how to find the volume of the cylinder(please add an explanation).

Answers

Explanation

From the image, we can see that the radius of the cylinder is given as

[tex]\frac{d}{2}=\frac{20}{2}=10ft[/tex]

The height of the cylinder is 40ft. Therefore, the volume of the cylinder is

[tex]volume=\pi r^2h=3.14\times10^2\times40=12560[/tex]

Answer: 12560 cubic feet

Which number is the same as (4-1)20-4-2), A - 2 B. 1/8C 2 D. 32 E. 512

Answers

[tex](4^{-1})(\frac{2^3}{4^{-2}})=4^{-1}\cdot4^2\cdot2^3=4\cdot2^3=4\cdot8=32[/tex]

1. Which of the following have the same domain and range? I. y = 2x - 1 II. y = -x + 2 x = 2 (A) I and III only (B) I and II only (C) II and III only (D) I, II and III

Answers

The I and II are linear function, so those have the same domain and the same range always. So the answer is B

Function F, shown below, assigns to a temperature given in degrees Celsius it's equivalent in degrees Fahrenheit. Function C, also shown below, assigns to a temperature given in degrees Kelvin its equivalent in degrees Celsius. Choose the response below that shows the correct expression for F(C(x)) and then choose the response below that correctly interprets the meaning of F(C(x)).F(x)=(9/5)x+32 C(x)=x−273

Answers

Given:

The expressions are given as,

[tex]\begin{gathered} F(x_)\text{ = \lparen}\frac{9}{5})x\text{ + 32} \\ C(x)\text{ = x - 273} \end{gathered}[/tex]

Required:

The value of F(C(x)) and its response.

Explanation:

The required function is calculated as,

[tex]\begin{gathered} F(C(x))\text{ = }\frac{9}{5}\text{ \lparen x - 273\rparen + 32} \\ F(C(x))\text{ = }\frac{9x}{5}\text{ - }\frac{2457}{5}\text{ + 32} \\ F(C(x))\text{ = }\frac{9x}{5}\text{ + 32 - }\frac{2457}{5} \\ F(C(x))\text{ = }\frac{9x}{5}\text{ + }\frac{160}{5}\text{ - }\frac{2457}{5} \\ F(C(x))\text{ = }\frac{9x}{5}\text{ - }\frac{2297}{5} \\ F(C(x))\text{ = }\frac{9x}{5}\text{ - 459}\frac{2}{5} \\ \end{gathered}[/tex]

assume you pay 300 per month into a retirement account for 12 years and they count has an APR of 3.05 compounded monthly question what is the account balance at the end of the 12 years round to the nearest cent

Answers

Given:

$300 per month in 12 years.

1 year = 12 months = 300 x 12 = $3600

For 12 years = $3600 x 12 = $43 200

Principal (P) = $43 200

rate(r) = 3.05% = 0.0305

Time (t) =12

n = 12

Using the formula below:

[tex]A=P(1+\frac{r}{n})^{nt}[/tex]

Substitute into the formula and evaluate

[tex]A=43200(1+\frac{0.0305}{12})^{12\times12}[/tex]

[tex]=43200(1+0.00254166667)^{144}[/tex][tex]=43200(1.00254166667)^{144}[/tex][tex]\approx62263.55[/tex]

Therefore, the account balance at the end of the 12 years is $62263.55

40% of x is 35.Write an equation that shows the relationship between 40%, x, and 35.My equation is: ___Use your equation to find x.x = ___

Answers

Answer:

E

87.5

Explanation:

40% of x is 35 can be expressed as an equation as shown below;

[tex]\begin{gathered} \frac{40}{100}\times x=35 \\ \end{gathered}[/tex]

Let's go ahead and solve for x;

[tex]\begin{gathered} \frac{40}{100}\times x=35 \\ \frac{4}{10}x=35 \\ 4x=350 \\ x=\frac{350}{4} \\ x=87.5 \end{gathered}[/tex]

What is the sale price of a $63 sweater if the discount rate is 15%?Round to the nearest cent. Do not put a $ in your answer.

Answers

Answer:

Concept:

The formula to calculate the selling price of the sweat will be

[tex]\text{Selling price=original price - discount}[/tex]

Step 1:

Calculate the discount price

The discount rate given is

[tex]=15\%[/tex]

The discounted price will be

[tex]\begin{gathered} =\frac{15}{100}\times\text{ \$63} \\ =\frac{945}{100} \\ =\text{ \$9.45} \end{gathered}[/tex]

Step 2:

Calculate the selling price, we will have

[tex]\begin{gathered} \text{Selling price=original price - discount} \\ \text{Selling price}=63-9.45 \\ \text{Selling price}=53.55 \end{gathered}[/tex]

Hence,

The sale price of the sweater will be = $ 53.55

Plot the points (-3,4) and (4,4) on the coordinate plane below.What is the distance between these two points?

Answers

To find the distance between two points A and B you can use the formula

[tex]\begin{gathered} d=\sqrt[]{(x_2-x_1)^2+(y_2-y_1)^2} \\ \text{ Where A and B have the coordinates} \\ A(x_1,y_1) \\ B(x_2,y_2) \end{gathered}[/tex]

So, in this case, you have

[tex]\begin{gathered} A(-3,4) \\ B(4,4) \end{gathered}[/tex][tex]\begin{gathered} d=\sqrt[]{(4_{}-(-3))^2+(4-4)^2} \\ d=\sqrt[]{(4_{}+3)^2+(0)^2} \\ d=\sqrt[]{(7)^2} \\ d=7 \end{gathered}[/tex]

Therefore, the distance between these points is 7 units.

Graphically,

Use the table below to answer the questions. a. Is this a proportional relationship? b. What is the constant of proportionality. X Y-2 6 1 -32 -6

Answers

hello

the relationship on the question is a direct proportion. i.e, when x in

n applicant receives a job offer from two different companies. Offer A is a starting salary of $58,000 and a 3% increase for 5 years. Offer B is a starting salary of $56,000 and an increase of $3,000 per year.

Answers

Part A.

The inital salary is $58,000, then we have:

[tex]a_1=58000_{}[/tex]

Since we have an increase of 3% each year we know that the second year the salary would be:

[tex]\begin{gathered} a_2=1.03a_1 \\ a_2=1.03\cdot58000 \end{gathered}[/tex]

The third year the salary would be:

[tex]\begin{gathered} a_3=1.03a_2 \\ a_3=1.03(1.03)58000 \\ a_3=(1.03)^258000 \end{gathered}[/tex]

and so on for year 4 and 5.

Since the increase in salary is only the first five years we conclude that this can't be represented by a geometric series.

For the first five year we can calculate the salary using a geometric sequence with common ratio 1.03, then for the first five years the salary is given by

[tex]a_n=(1.03)^{n-1}_{}\cdot58000\text{ for }1\leq n\leq5[/tex]

The salary for the any subsequent year is given by:

[tex]a_n=(1.03)^4\cdot58000\text{ for }n>5[/tex]

Part B.

Since we are adding a certain quantity each year we conclude that this offer can be represetend by an algebraic series given by:

[tex]\begin{gathered} b_n=56000+(n-1)3000 \\ b_n=56000+3000n-3000 \\ b_n=3000n+53000 \end{gathered}[/tex]

Part C.

After five years the income for offer A is:

[tex]a_5=(1.03)^4\cdot58000=65279.51[/tex]

For offer B is:

[tex]b_5=3000(5)+53000=68000_{}[/tex]

Therefore after 5 years job offer B has a greater total income.

-3x+5(x+2) find the equivalent

Answers

Answer

-3x + 5(x + 2) = 2x + 10

Explanation

The way to find this equivalent is simply to solve the given expression.

-3x + 5(x + 2)

= -3x + 5x + 10

= 2x + 10

Hope this Helps!!!

Hi I need help bad on this I have a report card Wednesday and my birthday is in 13 days

Answers

3)

- 18/(- 6)

This can be written as

- 18/- 6

Recall, if a negative number divides a negative number, the result is positive. Thus, the answer is 3

Complete the ratio table of the median price of renting a two-bedroom apartment by finding the value of x and y. Round answers to two decimal places. Norfolk $951 Richmond $1,042 1 X Х 100 у To solve the values set up and solve a a. Bar Chart b. Ratio c. Proportion d. Weighted Average x = y =

Answers

2+4=6 is true

7*8=56 is true

So the statement 2+4=6 AND 7*8=56 is also true

So the answer for question #36 is a) the statement is true because both proporsitions are true

In this figure, AB and CD are parallel.unitsAB is perpendicular to line segmentIf the length of EF is a units, then the length of GH is

Answers

AB is perpendicular to EF or GH because make a right angle

EF and GH have the samaevalue because they are between two paralells lines and make an right angle

Pat is walking to a restaurant 9 blocks away. Pat has walked 3 blocks in 7minutes. How many more minutes to reach the restaurant?

Answers

We can solve this question in the following way:

We have that Pat has walked 3 blocks in 7 minutes, and now, we can post the following: how long will it take to walk 9 blocks?

Then, we have:

Then, we can solve for x as follows:

We need to multiply 9 blocks by 7 minutes, and then we divide the result by 3 blocks (see the red lines).

[tex]x=\frac{9bl\cdot7\min}{3bl}\Rightarrow x=\frac{63\min}{3}\Rightarrow x=21\min [/tex]

This answer comes from having into account the following proportion:

[tex]\frac{9bl}{3bl}=\frac{x}{7\min}\Rightarrow x=\frac{9bl\cdot7\min}{3bl}=21\min [/tex]

(We have here proportional quantities.)

It will take 21 min to walk the 9 blocks. However, Pat has walked for 7 minutes. Therefore, Pat needs to walk for (21 min - 7 min) = 14 minutes.

Therefore, Pat will need to walk for 14 minutes more to reach the restaurant.

Answer:

15 minutes

Step-by-step explanation:

15 minutes

The blocks remaining is 7 - 2 = 5. The rate is (2 blocks)/(6 min).

So (2 blocks)/(6 min) = (5 blocks)/(x min).

2/6 = 5/x (units omitted for simplicity)

(2/6) × x = (5/x) × x (Multiplying both sides by x so x is not at the bottom).

(2/6) × x = 5

6 × (2/6) × x = 6 × 5

2x = 30

x = 15

Pat will take 15 minutes to walk the remaining 5 blocks.

a garden table and a bench cost $613 combined. the garden table cost $87 less then the bench. what is the cost of the bench ?

Answers

the cost of the bench is 350

Explanation

Step 1

let

x represents the cost of the garden table

y represents the cost of the bench

so

a)a garden table and a bench cost $613 combined.

[tex]x+y=613\rightarrow equation\text{ (1)}[/tex]

b)the garden table cost $87 less then the bench,( in other words, you have to add 87 to the price of the garden table to obtain the cost of the bech)

[tex]\begin{gathered} x+87=y\rightarrow Equation(2) \\ \end{gathered}[/tex]

Step 2

solve the equations

a)isolate x in equaiton (1) and replace in equation (2)

[tex]\begin{gathered} x+y=613\rightarrow equation\text{ (1)} \\ \text{subtrac y in both sides} \\ x+y-y=613-y \\ x=613-y \end{gathered}[/tex]

now, replace in equation (2)

[tex]\begin{gathered} x+87=y\rightarrow Equation(2) \\ (613-y)+87=y\rightarrow Equation(2) \\ 700-y=y \\ \text{add y in both sides} \\ 700-y+y=y+y \\ 700=2y \\ \text{divide both sides by 2} \\ \frac{700}{2}=\frac{2y}{2} \\ 350=y \end{gathered}[/tex]

it means, the cost of the bench is 350

Other Questions
In two days, Mariposa drinks seven 16-ouncebottles of water. She drinks the water in 4 equalservings. How many ounces of water doesMariposa drink in each serving? Write a two page story of your own that incorporates the style you have learned from reading The Merry Adventures of Robin Hood. The story should be based on Robin Hood, include a new adventure, and you are welcome to introduce a new character to the story as well. please help! 30 points Simplify: 6-(-9) divided by -9/-4 The formula S=C(1+r) models inflation, where C = the value today, r = the annual inflation rate, and S = the inflated value t years from now. a. If the inflation rate is 6%, how much will a house now worth $465,000 be worth in 10 years? b. If the inflation rate is 3%, how much will a house now worth $510,000 be worth in 5 years? Zero and negative exponentswrite in simplest for without zero or negative exponents10c - Step 1 Step 2 Step 3 Using the figures above, how many small squares will there be in step 4 and step 15? a. Step 4 = b. Step 15 = 2. A parallel-plate capacitor has an area of 2.0 cm, and the plates are separated by 2.0 mm. a. What is the capacitance? b. How much charge does this capacitor store when connected to a 6.0 V battery? a vector s has the initial point (-2,-4) and terminal point (-1,3) write s in the form s = ai + bj The flight of icarus. Review paragraphs 3-5. What elements of the setting help Daedalus carry out his plan? Please answer this question as fast as possible. Need answer as fraction or however it is, need asap In a bike race, a sport person starts his bike from rest and maintains a constant acceleration of 3 m/s2 for 8 seconds, and then a constant acceleration of 2 m/s2 for another 8 seconds. Determine the acquired velocity of the bike. For any right triangle, the side lengths of the triangle can be put in the equation a^2+ b^2 = c^2 where a, b, and c are the side lengths. A triangle with the side lengths 3 inches, 4 inches, and 5 inches is a right triangle. Which way(s) can you substitute the values into the equation to make it true? Which variable has to match the longest side length? Why? Which cell components are found in both plant and animal fill in the blank with the correct answer. the number _______is divisibel by 2, 3, 4, 5, and 6a)44b)180c)280d)385 question content area use the references to access important values if needed for this question. what atomic or hybrid orbitals make up the sigma bond between si and h in silane, sih4? orbital on si orbital on h what is the approximate h-si-h bond angle? Find each unknown function value or x value for f(x) = 4x - 7 and g(x) = -3x + 5 The two triangles below are similar. Also, m A = 15 and m ZC - 35 as shown below Find m2P, m 2Q, and m ZR. Assume the triangles are accurately drawn How is public health policy created? solve the system of linear equations by substitution 3y=-2x and y=x-5 Chlorine can bond with fluorine to form ClF. Chlorine can also bond with lithium to form LiCl. Which compound will have a greater partial charge?Question 2 options:A) Both compounds will have the same partial charge.B) ClFC) LiClD) Neither compound will have partial charge.